Prove that $int_1^inftyfracsin^2(x)xdx$ diverges. [on hold]Convergence tests for improper multiple integralsShow that the improper integral $int_1^infty f(x) dx$ exists iff $sum_1^infty a_n$ converges.Does the $n$th term test work for integrals?Is $int_1^inftyfracx cos(x)^21+x^3$ convergent or divergent?Show that $int_1^infty fracln xleft(1+x^2right)^lambdamathrm dx$ is convergent only for $lambda > frac12$Prove that the function $f(x)=fraccos^2xsqrtx^4+1$ is improperly integrable on $(0,infty)$.How to prove convergence of $int_0^inftyfracsin(x)xdx$ without evaluating itf is $C^1$ and decreasing then $int_1^inftyf(x)sin(x)dx$ converges.Finding the derivative of $f(x)=int_1^inftyfrace^-xyy^2dy,:::xin(0,infty)$improper integral problem check

What Happens when Passenger Refuses to Fly Boeing 737 Max?

Is "conspicuously missing" or "conspicuously" the subject of this sentence?

When a wind turbine does not produce enough electricity how does the power company compensate for the loss?

Are babies of evil humanoid species inherently evil?

How are showroom/display vehicles prepared?

Recommendation letter by significant other if you worked with them professionally?

Doesn't allowing a user mode program to access kernel space memory and execute the IN and OUT instructions defeat the purpose of having CPU modes?

Declaring and defining template, and specialising them

How to write ı (i without dot) character in pgf-pie

How does one describe somebody who is bi-racial?

Bash script should only kill those instances of another script's that it has launched

Does the nature of the Apocalypse in The Umbrella Academy change from the first to the last episode?

How can I ensure my trip to the UK will not have to be cancelled because of Brexit?

Reversed Sudoku

At what distance can a bugbear, holding a reach weapon, with the Polearm Master feat, get their Opportunity Attack?

In the late 1940’s to early 1950’s what technology was available that could melt a LOT of ice?

What wound would be of little consequence to a biped but terrible for a quadruped?

What was the Kree's motivation in Captain Marvel?

Should I tell my boss the work he did was worthless

When traveling to Europe from North America, do I need to purchase a different power strip?

Difference on montgomery curve equation between EFD and RFC7748

Why is computing ridge regression with a Cholesky decomposition much quicker than using SVD?

Can one live in the U.S. and not use a credit card?

What are some noteworthy "mic-drop" moments in math?



Prove that $int_1^inftyfracsin^2(x)xdx$ diverges. [on hold]


Convergence tests for improper multiple integralsShow that the improper integral $int_1^infty f(x) dx$ exists iff $sum_1^infty a_n$ converges.Does the $n$th term test work for integrals?Is $int_1^inftyfracx cos(x)^21+x^3$ convergent or divergent?Show that $int_1^infty fracln xleft(1+x^2right)^lambdamathrm dx$ is convergent only for $lambda > frac12$Prove that the function $f(x)=fraccos^2xsqrtx^4+1$ is improperly integrable on $(0,infty)$.How to prove convergence of $int_0^inftyfracsin(x)xdx$ without evaluating itf is $C^1$ and decreasing then $int_1^inftyf(x)sin(x)dx$ converges.Finding the derivative of $f(x)=int_1^inftyfrace^-xyy^2dy,:::xin(0,infty)$improper integral problem check













0












$begingroup$


Def: $int_1^infty f(x)dx:= lim_brightarrowinftyint_1^b f(x)dx$.



Motivation: Just a routine homework problem to check the student's understanding of comparison tests for convergence of improper integrals.



I gave my own answer below.










share|cite|improve this question









New contributor




Z Kane is a new contributor to this site. Take care in asking for clarification, commenting, and answering.
Check out our Code of Conduct.







$endgroup$



put on hold as off-topic by user21820, Cesareo, choco_addicted, Xander Henderson, Peter Foreman yesterday


This question appears to be off-topic. The users who voted to close gave this specific reason:


  • "This question is missing context or other details: Please provide additional context, which ideally explains why the question is relevant to you and our community. Some forms of context include: background and motivation, relevant definitions, source, possible strategies, your current progress, why the question is interesting or important, etc." – user21820, Cesareo, choco_addicted, Xander Henderson, Peter Foreman
If this question can be reworded to fit the rules in the help center, please edit the question.















  • $begingroup$
    According to Wolfram Alpha, it diverges.
    $endgroup$
    – Viktor Glombik
    2 days ago















0












$begingroup$


Def: $int_1^infty f(x)dx:= lim_brightarrowinftyint_1^b f(x)dx$.



Motivation: Just a routine homework problem to check the student's understanding of comparison tests for convergence of improper integrals.



I gave my own answer below.










share|cite|improve this question









New contributor




Z Kane is a new contributor to this site. Take care in asking for clarification, commenting, and answering.
Check out our Code of Conduct.







$endgroup$



put on hold as off-topic by user21820, Cesareo, choco_addicted, Xander Henderson, Peter Foreman yesterday


This question appears to be off-topic. The users who voted to close gave this specific reason:


  • "This question is missing context or other details: Please provide additional context, which ideally explains why the question is relevant to you and our community. Some forms of context include: background and motivation, relevant definitions, source, possible strategies, your current progress, why the question is interesting or important, etc." – user21820, Cesareo, choco_addicted, Xander Henderson, Peter Foreman
If this question can be reworded to fit the rules in the help center, please edit the question.















  • $begingroup$
    According to Wolfram Alpha, it diverges.
    $endgroup$
    – Viktor Glombik
    2 days ago













0












0








0


0



$begingroup$


Def: $int_1^infty f(x)dx:= lim_brightarrowinftyint_1^b f(x)dx$.



Motivation: Just a routine homework problem to check the student's understanding of comparison tests for convergence of improper integrals.



I gave my own answer below.










share|cite|improve this question









New contributor




Z Kane is a new contributor to this site. Take care in asking for clarification, commenting, and answering.
Check out our Code of Conduct.







$endgroup$




Def: $int_1^infty f(x)dx:= lim_brightarrowinftyint_1^b f(x)dx$.



Motivation: Just a routine homework problem to check the student's understanding of comparison tests for convergence of improper integrals.



I gave my own answer below.







integration analysis






share|cite|improve this question









New contributor




Z Kane is a new contributor to this site. Take care in asking for clarification, commenting, and answering.
Check out our Code of Conduct.











share|cite|improve this question









New contributor




Z Kane is a new contributor to this site. Take care in asking for clarification, commenting, and answering.
Check out our Code of Conduct.









share|cite|improve this question




share|cite|improve this question








edited yesterday







Z Kane













New contributor




Z Kane is a new contributor to this site. Take care in asking for clarification, commenting, and answering.
Check out our Code of Conduct.









asked 2 days ago









Z KaneZ Kane

285




285




New contributor




Z Kane is a new contributor to this site. Take care in asking for clarification, commenting, and answering.
Check out our Code of Conduct.





New contributor





Z Kane is a new contributor to this site. Take care in asking for clarification, commenting, and answering.
Check out our Code of Conduct.






Z Kane is a new contributor to this site. Take care in asking for clarification, commenting, and answering.
Check out our Code of Conduct.




put on hold as off-topic by user21820, Cesareo, choco_addicted, Xander Henderson, Peter Foreman yesterday


This question appears to be off-topic. The users who voted to close gave this specific reason:


  • "This question is missing context or other details: Please provide additional context, which ideally explains why the question is relevant to you and our community. Some forms of context include: background and motivation, relevant definitions, source, possible strategies, your current progress, why the question is interesting or important, etc." – user21820, Cesareo, choco_addicted, Xander Henderson, Peter Foreman
If this question can be reworded to fit the rules in the help center, please edit the question.







put on hold as off-topic by user21820, Cesareo, choco_addicted, Xander Henderson, Peter Foreman yesterday


This question appears to be off-topic. The users who voted to close gave this specific reason:


  • "This question is missing context or other details: Please provide additional context, which ideally explains why the question is relevant to you and our community. Some forms of context include: background and motivation, relevant definitions, source, possible strategies, your current progress, why the question is interesting or important, etc." – user21820, Cesareo, choco_addicted, Xander Henderson, Peter Foreman
If this question can be reworded to fit the rules in the help center, please edit the question.











  • $begingroup$
    According to Wolfram Alpha, it diverges.
    $endgroup$
    – Viktor Glombik
    2 days ago
















  • $begingroup$
    According to Wolfram Alpha, it diverges.
    $endgroup$
    – Viktor Glombik
    2 days ago















$begingroup$
According to Wolfram Alpha, it diverges.
$endgroup$
– Viktor Glombik
2 days ago




$begingroup$
According to Wolfram Alpha, it diverges.
$endgroup$
– Viktor Glombik
2 days ago










3 Answers
3






active

oldest

votes


















3












$begingroup$

hint



$$sin^2(x)=frac1-cos(2x)2$$



with



$$int_1^+inftyfraccos(2x)2xdx$$ convergent by Dirichlet' test and



$$int_1^+inftyfracdx2x$$
divergent.
the sum is divergent near $+infty$.






share|cite|improve this answer











$endgroup$








  • 1




    $begingroup$
    @ZKane it does not decompose into two divergent improper integrals. The integral of cos(2x)/2x is convergent!
    $endgroup$
    – Tomislav Ostojich
    2 days ago










  • $begingroup$
    @hamam_Abdallah Ok, thank you. I need to learn about Dirichlet's test for improper integrals. Do you happen to know where a proof of this result can be found?
    $endgroup$
    – Z Kane
    2 days ago










  • $begingroup$
    @ZKane Sorry, no. But stackexchange or google could help.
    $endgroup$
    – hamam_Abdallah
    2 days ago


















3












$begingroup$

Hint: $sin^2(x)$ is large when $x = k pi + fracpi 2$, which implies that the integral majorates the harmonic series.



Centered at every number of the form $x_k = k pi + fracpi 2$, we can find an interval $I_k$ of size $2delta$ (independent of $k$) such that $sin^2(x) geq 0.99$ for $x in I_k$.



Then $$int_1^infty fracsin^2 xx geq delta sum_k = 0^infty frac0.99k pi + fracpi2 = infty$$






share|cite|improve this answer









$endgroup$








  • 1




    $begingroup$
    (+1) Same idea but maybe easier, as it is more concrete: $sin^2 xge 1/2$ for $xin [pi/4 +kpi, 3pi/4 + kpi]$.
    $endgroup$
    – peter a g
    2 days ago



















0












$begingroup$

I think of an elementary way to show the crucial convergence of $int_1^infty fraccos(x)xdx$ without referring to the Dirichlet's test (although the idea is similar, i.e., integration by parts).



$$int_1^infty fraccos(t)tdt=fracsin(t)tbigg|_2^infty+int_2^infty fracsin(t)t^2dt$$



And the first term on the right is finite since



$$0leq bigg|fracsin(t)tbigg|leqbigg|frac1tbigg|rightarrow0 text as t rightarrowinfty $$



And the second term on the right converges for a similar reason



$$bigg|fracsin(t)t^2bigg|leqbigg|frac1t^2bigg|quadRightarrowquad int_2^infty bigg|fracsin(t)t^2bigg|dtleqint_2^infty frac1t^2dt=frac12$$






share|cite|improve this answer










New contributor




Z Kane is a new contributor to this site. Take care in asking for clarification, commenting, and answering.
Check out our Code of Conduct.






$endgroup$



















    3 Answers
    3






    active

    oldest

    votes








    3 Answers
    3






    active

    oldest

    votes









    active

    oldest

    votes






    active

    oldest

    votes









    3












    $begingroup$

    hint



    $$sin^2(x)=frac1-cos(2x)2$$



    with



    $$int_1^+inftyfraccos(2x)2xdx$$ convergent by Dirichlet' test and



    $$int_1^+inftyfracdx2x$$
    divergent.
    the sum is divergent near $+infty$.






    share|cite|improve this answer











    $endgroup$








    • 1




      $begingroup$
      @ZKane it does not decompose into two divergent improper integrals. The integral of cos(2x)/2x is convergent!
      $endgroup$
      – Tomislav Ostojich
      2 days ago










    • $begingroup$
      @hamam_Abdallah Ok, thank you. I need to learn about Dirichlet's test for improper integrals. Do you happen to know where a proof of this result can be found?
      $endgroup$
      – Z Kane
      2 days ago










    • $begingroup$
      @ZKane Sorry, no. But stackexchange or google could help.
      $endgroup$
      – hamam_Abdallah
      2 days ago















    3












    $begingroup$

    hint



    $$sin^2(x)=frac1-cos(2x)2$$



    with



    $$int_1^+inftyfraccos(2x)2xdx$$ convergent by Dirichlet' test and



    $$int_1^+inftyfracdx2x$$
    divergent.
    the sum is divergent near $+infty$.






    share|cite|improve this answer











    $endgroup$








    • 1




      $begingroup$
      @ZKane it does not decompose into two divergent improper integrals. The integral of cos(2x)/2x is convergent!
      $endgroup$
      – Tomislav Ostojich
      2 days ago










    • $begingroup$
      @hamam_Abdallah Ok, thank you. I need to learn about Dirichlet's test for improper integrals. Do you happen to know where a proof of this result can be found?
      $endgroup$
      – Z Kane
      2 days ago










    • $begingroup$
      @ZKane Sorry, no. But stackexchange or google could help.
      $endgroup$
      – hamam_Abdallah
      2 days ago













    3












    3








    3





    $begingroup$

    hint



    $$sin^2(x)=frac1-cos(2x)2$$



    with



    $$int_1^+inftyfraccos(2x)2xdx$$ convergent by Dirichlet' test and



    $$int_1^+inftyfracdx2x$$
    divergent.
    the sum is divergent near $+infty$.






    share|cite|improve this answer











    $endgroup$



    hint



    $$sin^2(x)=frac1-cos(2x)2$$



    with



    $$int_1^+inftyfraccos(2x)2xdx$$ convergent by Dirichlet' test and



    $$int_1^+inftyfracdx2x$$
    divergent.
    the sum is divergent near $+infty$.







    share|cite|improve this answer














    share|cite|improve this answer



    share|cite|improve this answer








    edited 2 days ago

























    answered 2 days ago









    hamam_Abdallahhamam_Abdallah

    38.2k21634




    38.2k21634







    • 1




      $begingroup$
      @ZKane it does not decompose into two divergent improper integrals. The integral of cos(2x)/2x is convergent!
      $endgroup$
      – Tomislav Ostojich
      2 days ago










    • $begingroup$
      @hamam_Abdallah Ok, thank you. I need to learn about Dirichlet's test for improper integrals. Do you happen to know where a proof of this result can be found?
      $endgroup$
      – Z Kane
      2 days ago










    • $begingroup$
      @ZKane Sorry, no. But stackexchange or google could help.
      $endgroup$
      – hamam_Abdallah
      2 days ago












    • 1




      $begingroup$
      @ZKane it does not decompose into two divergent improper integrals. The integral of cos(2x)/2x is convergent!
      $endgroup$
      – Tomislav Ostojich
      2 days ago










    • $begingroup$
      @hamam_Abdallah Ok, thank you. I need to learn about Dirichlet's test for improper integrals. Do you happen to know where a proof of this result can be found?
      $endgroup$
      – Z Kane
      2 days ago










    • $begingroup$
      @ZKane Sorry, no. But stackexchange or google could help.
      $endgroup$
      – hamam_Abdallah
      2 days ago







    1




    1




    $begingroup$
    @ZKane it does not decompose into two divergent improper integrals. The integral of cos(2x)/2x is convergent!
    $endgroup$
    – Tomislav Ostojich
    2 days ago




    $begingroup$
    @ZKane it does not decompose into two divergent improper integrals. The integral of cos(2x)/2x is convergent!
    $endgroup$
    – Tomislav Ostojich
    2 days ago












    $begingroup$
    @hamam_Abdallah Ok, thank you. I need to learn about Dirichlet's test for improper integrals. Do you happen to know where a proof of this result can be found?
    $endgroup$
    – Z Kane
    2 days ago




    $begingroup$
    @hamam_Abdallah Ok, thank you. I need to learn about Dirichlet's test for improper integrals. Do you happen to know where a proof of this result can be found?
    $endgroup$
    – Z Kane
    2 days ago












    $begingroup$
    @ZKane Sorry, no. But stackexchange or google could help.
    $endgroup$
    – hamam_Abdallah
    2 days ago




    $begingroup$
    @ZKane Sorry, no. But stackexchange or google could help.
    $endgroup$
    – hamam_Abdallah
    2 days ago











    3












    $begingroup$

    Hint: $sin^2(x)$ is large when $x = k pi + fracpi 2$, which implies that the integral majorates the harmonic series.



    Centered at every number of the form $x_k = k pi + fracpi 2$, we can find an interval $I_k$ of size $2delta$ (independent of $k$) such that $sin^2(x) geq 0.99$ for $x in I_k$.



    Then $$int_1^infty fracsin^2 xx geq delta sum_k = 0^infty frac0.99k pi + fracpi2 = infty$$






    share|cite|improve this answer









    $endgroup$








    • 1




      $begingroup$
      (+1) Same idea but maybe easier, as it is more concrete: $sin^2 xge 1/2$ for $xin [pi/4 +kpi, 3pi/4 + kpi]$.
      $endgroup$
      – peter a g
      2 days ago
















    3












    $begingroup$

    Hint: $sin^2(x)$ is large when $x = k pi + fracpi 2$, which implies that the integral majorates the harmonic series.



    Centered at every number of the form $x_k = k pi + fracpi 2$, we can find an interval $I_k$ of size $2delta$ (independent of $k$) such that $sin^2(x) geq 0.99$ for $x in I_k$.



    Then $$int_1^infty fracsin^2 xx geq delta sum_k = 0^infty frac0.99k pi + fracpi2 = infty$$






    share|cite|improve this answer









    $endgroup$








    • 1




      $begingroup$
      (+1) Same idea but maybe easier, as it is more concrete: $sin^2 xge 1/2$ for $xin [pi/4 +kpi, 3pi/4 + kpi]$.
      $endgroup$
      – peter a g
      2 days ago














    3












    3








    3





    $begingroup$

    Hint: $sin^2(x)$ is large when $x = k pi + fracpi 2$, which implies that the integral majorates the harmonic series.



    Centered at every number of the form $x_k = k pi + fracpi 2$, we can find an interval $I_k$ of size $2delta$ (independent of $k$) such that $sin^2(x) geq 0.99$ for $x in I_k$.



    Then $$int_1^infty fracsin^2 xx geq delta sum_k = 0^infty frac0.99k pi + fracpi2 = infty$$






    share|cite|improve this answer









    $endgroup$



    Hint: $sin^2(x)$ is large when $x = k pi + fracpi 2$, which implies that the integral majorates the harmonic series.



    Centered at every number of the form $x_k = k pi + fracpi 2$, we can find an interval $I_k$ of size $2delta$ (independent of $k$) such that $sin^2(x) geq 0.99$ for $x in I_k$.



    Then $$int_1^infty fracsin^2 xx geq delta sum_k = 0^infty frac0.99k pi + fracpi2 = infty$$







    share|cite|improve this answer












    share|cite|improve this answer



    share|cite|improve this answer










    answered 2 days ago









    rabotarabota

    14.2k32782




    14.2k32782







    • 1




      $begingroup$
      (+1) Same idea but maybe easier, as it is more concrete: $sin^2 xge 1/2$ for $xin [pi/4 +kpi, 3pi/4 + kpi]$.
      $endgroup$
      – peter a g
      2 days ago













    • 1




      $begingroup$
      (+1) Same idea but maybe easier, as it is more concrete: $sin^2 xge 1/2$ for $xin [pi/4 +kpi, 3pi/4 + kpi]$.
      $endgroup$
      – peter a g
      2 days ago








    1




    1




    $begingroup$
    (+1) Same idea but maybe easier, as it is more concrete: $sin^2 xge 1/2$ for $xin [pi/4 +kpi, 3pi/4 + kpi]$.
    $endgroup$
    – peter a g
    2 days ago





    $begingroup$
    (+1) Same idea but maybe easier, as it is more concrete: $sin^2 xge 1/2$ for $xin [pi/4 +kpi, 3pi/4 + kpi]$.
    $endgroup$
    – peter a g
    2 days ago












    0












    $begingroup$

    I think of an elementary way to show the crucial convergence of $int_1^infty fraccos(x)xdx$ without referring to the Dirichlet's test (although the idea is similar, i.e., integration by parts).



    $$int_1^infty fraccos(t)tdt=fracsin(t)tbigg|_2^infty+int_2^infty fracsin(t)t^2dt$$



    And the first term on the right is finite since



    $$0leq bigg|fracsin(t)tbigg|leqbigg|frac1tbigg|rightarrow0 text as t rightarrowinfty $$



    And the second term on the right converges for a similar reason



    $$bigg|fracsin(t)t^2bigg|leqbigg|frac1t^2bigg|quadRightarrowquad int_2^infty bigg|fracsin(t)t^2bigg|dtleqint_2^infty frac1t^2dt=frac12$$






    share|cite|improve this answer










    New contributor




    Z Kane is a new contributor to this site. Take care in asking for clarification, commenting, and answering.
    Check out our Code of Conduct.






    $endgroup$

















      0












      $begingroup$

      I think of an elementary way to show the crucial convergence of $int_1^infty fraccos(x)xdx$ without referring to the Dirichlet's test (although the idea is similar, i.e., integration by parts).



      $$int_1^infty fraccos(t)tdt=fracsin(t)tbigg|_2^infty+int_2^infty fracsin(t)t^2dt$$



      And the first term on the right is finite since



      $$0leq bigg|fracsin(t)tbigg|leqbigg|frac1tbigg|rightarrow0 text as t rightarrowinfty $$



      And the second term on the right converges for a similar reason



      $$bigg|fracsin(t)t^2bigg|leqbigg|frac1t^2bigg|quadRightarrowquad int_2^infty bigg|fracsin(t)t^2bigg|dtleqint_2^infty frac1t^2dt=frac12$$






      share|cite|improve this answer










      New contributor




      Z Kane is a new contributor to this site. Take care in asking for clarification, commenting, and answering.
      Check out our Code of Conduct.






      $endgroup$















        0












        0








        0





        $begingroup$

        I think of an elementary way to show the crucial convergence of $int_1^infty fraccos(x)xdx$ without referring to the Dirichlet's test (although the idea is similar, i.e., integration by parts).



        $$int_1^infty fraccos(t)tdt=fracsin(t)tbigg|_2^infty+int_2^infty fracsin(t)t^2dt$$



        And the first term on the right is finite since



        $$0leq bigg|fracsin(t)tbigg|leqbigg|frac1tbigg|rightarrow0 text as t rightarrowinfty $$



        And the second term on the right converges for a similar reason



        $$bigg|fracsin(t)t^2bigg|leqbigg|frac1t^2bigg|quadRightarrowquad int_2^infty bigg|fracsin(t)t^2bigg|dtleqint_2^infty frac1t^2dt=frac12$$






        share|cite|improve this answer










        New contributor




        Z Kane is a new contributor to this site. Take care in asking for clarification, commenting, and answering.
        Check out our Code of Conduct.






        $endgroup$



        I think of an elementary way to show the crucial convergence of $int_1^infty fraccos(x)xdx$ without referring to the Dirichlet's test (although the idea is similar, i.e., integration by parts).



        $$int_1^infty fraccos(t)tdt=fracsin(t)tbigg|_2^infty+int_2^infty fracsin(t)t^2dt$$



        And the first term on the right is finite since



        $$0leq bigg|fracsin(t)tbigg|leqbigg|frac1tbigg|rightarrow0 text as t rightarrowinfty $$



        And the second term on the right converges for a similar reason



        $$bigg|fracsin(t)t^2bigg|leqbigg|frac1t^2bigg|quadRightarrowquad int_2^infty bigg|fracsin(t)t^2bigg|dtleqint_2^infty frac1t^2dt=frac12$$







        share|cite|improve this answer










        New contributor




        Z Kane is a new contributor to this site. Take care in asking for clarification, commenting, and answering.
        Check out our Code of Conduct.









        share|cite|improve this answer



        share|cite|improve this answer








        edited 2 days ago





















        New contributor




        Z Kane is a new contributor to this site. Take care in asking for clarification, commenting, and answering.
        Check out our Code of Conduct.









        answered 2 days ago









        Z KaneZ Kane

        285




        285




        New contributor




        Z Kane is a new contributor to this site. Take care in asking for clarification, commenting, and answering.
        Check out our Code of Conduct.





        New contributor





        Z Kane is a new contributor to this site. Take care in asking for clarification, commenting, and answering.
        Check out our Code of Conduct.






        Z Kane is a new contributor to this site. Take care in asking for clarification, commenting, and answering.
        Check out our Code of Conduct.













            Popular posts from this blog

            Lowndes Grove History Architecture References Navigation menu32°48′6″N 79°57′58″W / 32.80167°N 79.96611°W / 32.80167; -79.9661132°48′6″N 79°57′58″W / 32.80167°N 79.96611°W / 32.80167; -79.9661178002500"National Register Information System"Historic houses of South Carolina"Lowndes Grove""+32° 48' 6.00", −79° 57' 58.00""Lowndes Grove, Charleston County (260 St. Margaret St., Charleston)""Lowndes Grove"The Charleston ExpositionIt Happened in South Carolina"Lowndes Grove (House), Saint Margaret Street & Sixth Avenue, Charleston, Charleston County, SC(Photographs)"Plantations of the Carolina Low Countrye

            random experiment with two different functions on unit interval Announcing the arrival of Valued Associate #679: Cesar Manara Planned maintenance scheduled April 23, 2019 at 00:00UTC (8:00pm US/Eastern)Random variable and probability space notionsRandom Walk with EdgesFinding functions where the increase over a random interval is Poisson distributedNumber of days until dayCan an observed event in fact be of zero probability?Unit random processmodels of coins and uniform distributionHow to get the number of successes given $n$ trials , probability $P$ and a random variable $X$Absorbing Markov chain in a computer. Is “almost every” turned into always convergence in computer executions?Stopped random walk is not uniformly integrable

            How should I support this large drywall patch? Planned maintenance scheduled April 23, 2019 at 00:00UTC (8:00pm US/Eastern) Announcing the arrival of Valued Associate #679: Cesar Manara Unicorn Meta Zoo #1: Why another podcast?How do I cover large gaps in drywall?How do I keep drywall around a patch from crumbling?Can I glue a second layer of drywall?How to patch long strip on drywall?Large drywall patch: how to avoid bulging seams?Drywall Mesh Patch vs. Bulge? To remove or not to remove?How to fix this drywall job?Prep drywall before backsplashWhat's the best way to fix this horrible drywall patch job?Drywall patching using 3M Patch Plus Primer